Prove a Floor Inequality

During a math class, a proposed problem on a handout is as follows:

Prove that \[\lfloor x\rfloor +\dfrac{\lfloor 2x\rfloor}{2}+\dfrac{\lfloor 3x\rfloor}{3}+\cdots +\dfrac{\lfloor nx\rfloor}{n}\le \lfloor nx\rfloor\] for all positive reals \(x\) and integer \(n\).

Can someone help me prove it? I tried to use a periodic argument, proving it true for x[0,1)x\in [0,1) then plugging in x=x+kx=x'+k for positive integer kk to prove the inequality for all other xx. However it didn't work, because my inequality substitutions were too strict and I ended up with (n1)xnx(n-1)x\le \lfloor nx\rfloor which is not true.

Thanks!

#Algebra #HelpMe! #Inequality #FloorFunction #Proof

Note by Daniel Liu
7 years ago

No vote yet
1 vote

  Easy Math Editor

This discussion board is a place to discuss our Daily Challenges and the math and science related to those challenges. Explanations are more than just a solution — they should explain the steps and thinking strategies that you used to obtain the solution. Comments should further the discussion of math and science.

When posting on Brilliant:

  • Use the emojis to react to an explanation, whether you're congratulating a job well done , or just really confused .
  • Ask specific questions about the challenge or the steps in somebody's explanation. Well-posed questions can add a lot to the discussion, but posting "I don't understand!" doesn't help anyone.
  • Try to contribute something new to the discussion, whether it is an extension, generalization or other idea related to the challenge.
  • Stay on topic — we're all here to learn more about math and science, not to hear about your favorite get-rich-quick scheme or current world events.

MarkdownAppears as
*italics* or _italics_ italics
**bold** or __bold__ bold

- bulleted
- list

  • bulleted
  • list

1. numbered
2. list

  1. numbered
  2. list
Note: you must add a full line of space before and after lists for them to show up correctly
paragraph 1

paragraph 2

paragraph 1

paragraph 2

[example link](https://brilliant.org)example link
> This is a quote
This is a quote
    # I indented these lines
    # 4 spaces, and now they show
    # up as a code block.

    print "hello world"
# I indented these lines
# 4 spaces, and now they show
# up as a code block.

print "hello world"
MathAppears as
Remember to wrap math in \( ... \) or \[ ... \] to ensure proper formatting.
2 \times 3 2×3 2 \times 3
2^{34} 234 2^{34}
a_{i-1} ai1 a_{i-1}
\frac{2}{3} 23 \frac{2}{3}
\sqrt{2} 2 \sqrt{2}
\sum_{i=1}^3 i=13 \sum_{i=1}^3
\sin \theta sinθ \sin \theta
\boxed{123} 123 \boxed{123}

Comments

http://mks.mff.cuni.cz/kalva/usa/usoln/usol815.html

mathh mathh - 7 years ago

Apparently this is USAMO 1981 #5.

David Lee - 7 years ago

Log in to reply

Yep.

Daniel Liu - 7 years ago

Log in to reply

I was just gonna say...

Kaan Dokmeci - 7 years ago

I think using induction approach should also work

Roger Lu - 7 years ago

I don't know if I am right or wrong ... So please correct me ... Thanks in advance ... here is my Solution : As we know, floor(x) = x - {x} , where {x} is fractional part. if we replace all the floor values in the above eqn will reduce to

LHS : nx - [ {x} + ({2x}/2) + ({3x}/3) + ({4x}/4) + ........... + ({nx}/n) ]

What is interesting to see is that {x} = {2x}/2 = {3x}/3 = ..... = {nx}/n (You can check it with any value of x :) )

Hence LHS will become : nx - n{x}

Solving RHS we got : floor(nx) = nx - {nx} As the fractional part of {nx} will always lie between 0 and 1, i.e. 0 <= {nx} < 1

And it is easy to see that the product of n and {x} in LHS is always greater than or equal to (in case of 0 as fractional value) {nx} in RHS.

Probably this will prove it ... What say guys ???

Vipin Kumar - 6 years, 12 months ago

Log in to reply

@Daniel Liu @Finn Hulse : Guys can you please check the above solution ???

Vipin Kumar - 6 years, 12 months ago

Log in to reply

I still don't think it proves it. @Finn Hulse

Your solution depends on the fact that {x}+{2x}2++{nx}n{nx}\{x\}+\dfrac{\{2x\}}{2}+\cdots+\dfrac{\{nx\}}{n}\ge \{nx\} but you didn't prove that.

Daniel Liu - 6 years, 11 months ago

Log in to reply

@Daniel Liu Meh I'm tired. :P

Finn Hulse - 6 years, 11 months ago

It's good as far as I can tell! :D

Finn Hulse - 6 years, 12 months ago

n{x}{nx}n\{x\}\neq \{nx\}. Try, e.g., x=0.6x=0.6, n=2n=2. Then 2{0.6}=1.2{20.6}=0.22\cdot \{0.6\}=1.2\neq\{2\cdot 0.6\}=0.2.

mathh mathh - 6 years, 12 months ago

Log in to reply

When did I say n{x} = {nx} ??? Please check it once more :)

Vipin Kumar - 6 years, 12 months ago

Log in to reply

@Vipin Kumar "What is interesting to see is that {x} = {2x}/2 = {3x}/3 = ..... = {nx}/n"

I could have equally said that {x}{nx}n\{x\}\neq \frac{\{nx\}}{n}, just multiply both sides of your equality by nn (we can, since n0n\neq 0). Both of the equalities are equivalent.

mathh mathh - 6 years, 12 months ago

Log in to reply

@Mathh Mathh Yups got my mistake... thank you... :)

Vipin Kumar - 6 years, 12 months ago

that should prove it man :)...... i don't find any serious anomaly..... good job

Abhinav Raichur - 6 years, 12 months ago

Log in to reply

{x} = {2x}/2 = {3x}/3 = ..... = {nx}/n is false (see my other comments).

mathh mathh - 6 years, 12 months ago

Log in to reply

@Mathh Mathh yes and it is just the special case where inequality turns to equality...... it gives maximum value of LHS

Abhinav Raichur - 6 years, 12 months ago
×

Problem Loading...

Note Loading...

Set Loading...